Đến nội dung

iloveyouproht nội dung

Có 153 mục bởi iloveyouproht (Tìm giới hạn từ 30-04-2020)



Sắp theo                Sắp xếp  

#662905 gõ thử công thức toán

Đã gửi bởi iloveyouproht on 24-11-2016 - 15:44 trong Thử các chức năng của diễn đàn

Bất đẳng thức tương đương :
 
$1+\frac{4\sum ab}{3+\sum ab} \ge 3\sqrt[3]{ \frac{8abc}{(a+1)(b+1)(c+1)}}$
Ta có :
 
$1+\frac{4\sum ab}{3+\sum ab} \ge 3 \sqrt[3]{\frac{4(\sum ab)^2}{(3+\sum ab)^2}}$. Đưa về cm :
 
$(\sum ab )^2 (a+1)(b+1)(c+1) \ge 2abc (3+\sum ab )^2$
 
$\Leftrightarrow (\sum ab )^2 ( \sum ab -abc +4)\ge 6abc(3+2\sum ab )$
 
mà $ (\sum ab )^2  \ge 3abc(a+b+c)=9abc$,=>
 
$ 3 ( \sum ab -abc +4)\ge 2 (3+2\sum ab )$
 
$\Leftrightarrow 12 \ge 3abc+2\sum ab$ ( right ) => 



#653973 gõ thử công thức toán

Đã gửi bởi iloveyouproht on 12-09-2016 - 23:37 trong Thử các chức năng của diễn đàn

Trước hết, ta chứng minh BĐT sau :
$$\dfrac{1}{\sqrt{a^2+bc}}+\dfrac{1}{\sqrt{b^2+ca}}+\dfrac{1}{\sqrt{c^2+ab}} \le \sqrt{2}\left (\dfrac{1}{a+b}+\dfrac{1}{b+c}+\dfrac{1}{c+a}\right )$$

Áp dụng CS, ta có :
$$\left (\sum \dfrac{1}{\sqrt{a^2+bc}}\right )^2 =\left [\sum \sqrt{\dfrac{(a+b)(a+c)}{a^2+bc}}.\dfrac{1}{\sqrt{(a+b)(a+c)}}\right ]^2 \le \left [\sum \dfrac{(a+b)(a+c)}{a^2+bc}\right ]\left [\sum \dfrac{1}{(a+b)(a+c)}\right ]$$
$$=\dfrac{2(a+b+c)}{(a+b)(b+c)(c+a)}.\left [\sum \dfrac{a(b+c)}{a^2+bc}+3\right ]$$
Như vậy, chỉ cần chứng minh :
$$\dfrac{2(a+b+c)}{(a+b)(b+c)(c+a)}.\left [\sum \dfrac{a(b+c)}{a^2+bc}+3\right ]\le 2\left (\sum \dfrac{1}{b+c}\right )^2$$
$$\Leftrightarrow \sum \dfrac{a(b+c)}{a^2+bc}+3\le \dfrac{\left (a^2+b^2+c^2+3ab+3bc+3ca\right )^2}{(a+b)(b+c)(c+a)(a+b+c)}$$
$$\Leftrightarrow \sum \dfrac{a(b+c)}{a^2+bc}-3 \le \dfrac{a^4+b^4+c^4-a^2b^2-b^2c^2-c^2a^2}{(a+b)(b+c)(c+a)(a+b+c)}$$
$$\Leftrightarrow \sum (a-b)(a-c)\left [\dfrac{1}{a^2+bc}+\dfrac{1}{(b+c)(a+b+c)}\right ] \ge 0$$
Không mất tính tổng quát, giả sử $a\ge b\ge c$, khi đó, ta có $a-c\ge \dfrac{a}{b}(b-c) \ge 0$
Do đó :
$$ \sum (a-b)(a-c)\left [\dfrac{1}{a^2+bc}+\dfrac{1}{(b+c)(a+b+c)}\right ] \ge \dfrac{(a-b)(a-c)}{b}\left \{a\left [\dfrac{1}{a^2+bc}+\dfrac{1}{(b+c)(a+b+c)}\right ]-b\left [\dfrac{1}{b^2+ca}+\dfrac{1}{(a+c)(a+b+c)}\right ]\right \}$$
$$=\dfrac{c(a-b)^2(a+b)(b-c)\left (a^2+b^2-ab+ac+bc\right )}{b(a+c)(b+c)(\left (a^2+bc\right )\left (b^2+ca\right )} \ge 0$$
Trở lại bài toán, ta chỉ cần chứng minh :
$$\sqrt{2}\left (\dfrac{1}{a+b}+\dfrac{1}{b+c}+\dfrac{1}{c+a}\right ) \le \dfrac{3(a+b+c)}{\sqrt{2}(ab+bc+ca)}$$
Thật vậy :
$$\dfrac{1}{a+b}+\dfrac{1}{b+c}+\dfrac{1}{c+a}\ =\dfrac{a^2+b^2+c^2+3(ab+bc+ca)}{(a+b)(b+c)(c+a)}$$
$$\le \dfrac{(a+b+c)^2+ab+bc+ca}{8\dfrac{(a+b+c)(ab+bc+ca)}{9}} \le \dfrac{3}{2}\dfrac{a+b+c}{ab+bc+ca}$$
$$\Leftrightarrow \sqrt{2}\left (\dfrac{1}{a+b}+\dfrac{1}{b+c}+\dfrac{1}{c+a}\right ) \le \dfrac{3(a+b+c)}{\sqrt{2}(ab+bc+ca)}$$
BĐT đã được chứng minh. Đẳng thức xảy ra khi $a=b=c$.

Qúy <3 Kiên




#652509 gõ thử công thức toán

Đã gửi bởi iloveyouproht on 02-09-2016 - 23:42 trong Thử các chức năng của diễn đàn

5a . Đầu tiên ta chứng minh P $\leq 3$

    

Ta có : P-3=$\sum \frac{(a+b)^{2}}{\sum a^{2}+ab} -3 =\sum \frac{ab-c^{2}}{\sum a^{2}+ab} =\frac{(ab-c^{2})(3+\frac{1}{ab})}{(\sum a^{2}+ab)(1+1+1+\frac{1}{ab})} \leq \frac{3(\sum ab) -3(\sum c^{2})-\sum \frac{c^{2}}{ab}}{(a+b+c+1)^{2}}$ 

=> P-3 $\leq \frac{(a+b+c)^{2}+3-(a+b+c)^{2}-3}{(a+b+c+1)^{2}}=0$

=> P $\leq 3$ 

Dấu = tại a=b=c

Tiếp đến sẽ cm $\geq 2$ 

Ta có : $\sum \frac{(a+b)^{2}}{a^{2}+b^{2}+c^{2}+ab}\geq \sum \frac{(a+b)^{2}}{\sum a^{2}+\sum ab }=\frac{\sum (a+b)^{2}}{\sum a^{2}+\sum ab}=2$

Dấu = tại a=b=0 




#634597 Cho 3 số dương a,b,c. Chứng minh:$\sqrt{\frac{a...

Đã gửi bởi iloveyouproht on 21-05-2016 - 22:51 trong Bất đẳng thức và cực trị

Đã trả lời tại đây : http://diendantoanho...-2/#entry633507




#664734 Tìm GTNN của biểu thức: $\sqrt{a^2+ab+2b^2} +\sqrt{b^2+bc+2c^...

Đã gửi bởi iloveyouproht on 15-12-2016 - 20:17 trong Bất đẳng thức và cực trị

Cho a, b, c là các số thực không âm thỏa mãn điều kiện a + b + c = 1. Tìm GTNN của biểu thức: 

$\sqrt{a^2+ab+2b^2} +\sqrt{b^2+bc+2c^2} +\sqrt{c^2+ac+2a^2}$.

mong mọi người giải giùm.

Ở đây mình có cách giải tổng quát bài này chỉ bằng phép biến đổi tương đương . B xem tại đây : http://diendantoanho...ca2a2geq-sqrt5/




#656675 $\sum \frac{4}{a+b} \leq \frac{1}{a}+\frac{1}{b...

Đã gửi bởi iloveyouproht on 04-10-2016 - 17:00 trong Bất đẳng thức và cực trị

Bài 1: Cho $a,b,c$ là các số thực dương thoả mãn $abc = 1$.Chứng minh rằng:

        $a^4 + b^4 +b^4 + a + b + c + \frac{2a}{b^2+c^2} + \frac{2b}{a^2+c^2} + \frac{2c}{a^2+b^2} \geq 9$

Bài 2: Cho $a,b,c$ là các số thực dương tuỳ ý. Chứng minh rằng:

        $\sqrt{5a^2+4bc} + \sqrt{5b^2+4ca} + \sqrt{5c^2+4ab} \geq \sqrt{3(a^2+b^2+c^2)} + 2(\sqrt{ab}+\sqrt{bc}+\sqrt{ca})$

Bài 3: Cho $a,b,c$ là các số thực dương thoả mãn $a+b+c = 1$. Chứng minh rằng:

        $\frac{4}{a+b}+\frac{4}{b+c}+\frac{4}{c+a} \leq \frac{1}{a}+\frac{1}{b}+\frac{1}{c}+9$

Bài 4: Cho $a,b,c$ là các số thực dương thoả mãn $ab+bc+ca = 1$. Chứng minh rằng:

        $\frac{a^3}{1+9b^2ca}+\frac{b^3}{1+9c^2ab}+\frac{c^3}{1+9a^2bc} \geq \frac{(a+b+c)^3}{18}$

Bài 5: Cho $a,b,c$ phân biệt. Chứng minh rằng: 

        $(a^2+b^2+c^2)\left ( \frac{1}{(a-b)^2}+\frac{1}{(b-c)^2}+\frac{1}{(c-a)^2}\right ) \geq \frac{9}{2}$

Bài 6: Cho $a,b,c$ là các số thực dương thoả mãn $2ab+6bc+2ac=7abc$

     Tìm GTNN của $C=\frac{4ab}{a+2b}+\frac{9ca}{a+4c}+\frac{4bc}{b+c}$

Bài 7: Cho $a,b,c$ là các số thực dương thoả mãn $2\left ( \frac{a}{b}+\frac{b}{a}\right )+c\left ( \frac{a}{b^2}+\frac{b}{a^2} \right )=6$.

     Tìm GTNN của $P=\frac{bc}{a(2b+c)}+\frac{ca}{b(2a+c)}+\frac{4ab}{c(a+b)}$

Bài 8: Cho 3 sô thực dương $x,y,z$ thoả mãn $x+y+z \leq \frac{3}{2}$.

     Tìm GTNN của $P=\frac{x(yz+1)^2}{z^2(zx+1)}+\frac{y(zx+1)^2}{x^2(xy+1)}+\frac{z(xy+1)^2}{y^2(yz+1)}$

Bài 9: Cho các số thực dương thoả mãn $\sqrt{a}+\sqrt{b}+\sqrt{c}=1$.

     Tìm GTNN của $P=\sqrt{2a^2+ab+2b^2}+\sqrt{2b^2+bc+2c^2}+\sqrt{2c^2+ca+2a^2}$.

Mọi người giúp em với ạ. Em xin cảm ơn!!!

Bài 2 đã được anh dogsteven Giari . Mình xin trích lại như sau : 

Bất đẳng thức có tích rời rạc, việc đầu tiên của ta là gom lại.

Bất đẳng thức trên tương đương với: $\sum \dfrac{5a^2}{\sqrt{5a^2+4bc}+2\sqrt{bc}}\geqslant \sqrt{3(a^2+b^2+c^2)}$

Áp dụng bất đẳng thức Cauchy-Schwarz: $VT\geqslant \dfrac{5(a^2+b^2+c^2)^2}{\sum a^2\sqrt{5a^2+4bc}+2\sum a^2\sqrt{bc}}$

Tiếp theo là "phá căn". Áp dụng bất đẳng thức Cauchy-Schwarz:

$\sum a^2\sqrt{5a^2+4bc}\leqslant \sqrt{(a^2+b^2+c^2)\left[5(a^4+b^4+c^4)+4abc(a+b+c)\right]}$

$2\sum a^2\sqrt{bc}\leqslant \dfrac{2}{\sqrt{3}}\sqrt{a^2+b^2+c^2}(ab+bc+ca)$

Do đó ta chỉ cần chứng minh: $5(a^2+b^2+c^2)\geqslant \sqrt{15(a^4+b^4+c^4)+12(ab+bc+ca)}+2(ab+bc+ca)$

Đến đây dễ rồi.




#660925 $a^{3} + b^{3} + c^{3} + 6abc \geq 9...

Đã gửi bởi iloveyouproht on 07-11-2016 - 00:16 trong Bất đẳng thức và cực trị

Cho a, b, c là các số thực không âm thỏa mãn điều kiện ab + bc + ca   = 3. Chứng minh rằng

$a^{3} + b^{3} + c^{3} + 6abc \geq 9$

Ta có : P= $a^{3} + b^{3} + c^{3} + 6abc$ => 2P =  $2(a^{3} + b^{3} + c^{3}) + 12abc$

Mà : $a^{3} + a^{3} + 4 \geq 6a^{2}$

Tương tự với b,c cộng lại ta được : $2P \geq  6(a^{2} + b^{2} + c^{2} + 12abc-12$ 

Ta có bđt : $a^{2} + b^{2} + c^{2} + 2abc \geq 2 ( ab + bc +ca )$ ( schur ) 

=> $2P \geq  6(a^{2} + b^{2} + c^{2} + 12abc-12 \geq 12( ab +bc +ca )-12=18$

=> đpcm 




#687228 Cho $x,y,z > 0$ thỏa xyz=1. Chứng minh rằng: $\sum...

Đã gửi bởi iloveyouproht on 11-07-2017 - 15:16 trong Bất đẳng thức và cực trị

Bài 1. Cho $x,y,z > 0$ thỏa xyz=1. Chứng minh rằng: $\sum \frac{1}{1+x+x^2} \geq  1$

Bài 2. Cho a, b , c > 0 thỏa $a+b+c =1$. Chứng minh rằng 

$\frac{a^2+b}{b+c} + \frac{b^2+c}{c+a} + \frac{c^2+a}{a+b} \geq 2$

Bài 3. Cho a, b, c > 0  thỏa $a^2+b^2+c^2 = 3$. Chứng minh rằng: $\sum \frac{1}{2-a}\geq 3$

1.

WAK3jTe.png

btPsxx9.png




#675525 Chứng minh rằng : $3(a^{2}+b^{2}+c^{2})+abc \geq 10$

Đã gửi bởi iloveyouproht on 28-03-2017 - 15:26 trong Bất đẳng thức và cực trị

Cho a,b,c là các số thực dương thỏa mãn $ab+bc+ca=3.$

Chứng minh rằng : $3(a^{2}+b^{2}+c^{2})+abc \geq 10$

Ta có bđt phụ : $\sum a^{2}+2abc+1\geq 2(\sum ab)$ (*)

Chứng minh (*): https://diendantoanh...bc1geq-2abbcca/

Áp dụng bất đẳng thức (*) ta có : $3(\sum a^{2})+abc=\frac{1}{2}\left [ 6(\sum a^{2})+2abc \right ]\geq \frac{1}{2}\left [ 5(\sum a^{2})+2(\sum ab) -1\right ]\geq \frac{1}{2}\left [ 7(\sum ab) -1\right ]=10$

-




#675710 Chứng minh rằng : $3(a^{2}+b^{2}+c^{2})+abc \geq 10$

Đã gửi bởi iloveyouproht on 30-03-2017 - 14:14 trong Bất đẳng thức và cực trị

đề là ab+bc+ca+abc=4 ạ, mình viết nhầm nữa ._.

Nếu gt thay đổi thì ta cũng chỉ cần biến đổi thêm chút là đc :v

Từ gt=>abc$\leq 1$

Ta có bđt phụ : $\sum a^{2}+2abc+1\geq 2(\sum ab)$ (*)

Chứng minh (*): https://diendantoanh...bc1geq-2abbcca/

Áp dụng bất đẳng thức (*) ta có : $3(\sum a^{2})+abc=\frac{1}{2}\left [ 6(\sum a^{2})+2abc \right ]\geq \frac{1}{2}\left [ 5(\sum a^{2})+2(\sum ab) -1\right ]\geq \frac{1}{2}\left [ 7(\sum ab) -1\right ]= \frac{1}{2}\left [ 7(4-abc) -1\right ]\geq 10$




#672624 Cho a,b,c >0 Tìm min $$\sum \frac{a}{a...

Đã gửi bởi iloveyouproht on 24-02-2017 - 19:55 trong Bất đẳng thức và cực trị

2.    $\frac{1}{a(1+b)}+\frac{1}{b(1+c)}+\frac{1}{c(1+a)}\geq \frac{3}{1+abc}$ vs a,b,c>0

cách khác nếu b cần :)

0rSm0oO.png




#639011 Tìm max $(1+2a)(1+2bc)$

Đã gửi bởi iloveyouproht on 08-06-2016 - 21:23 trong Bất đẳng thức và cực trị

Đúng r . Cho xin luôn :)




#647052 Cho x, y, z>0 : CMR: $\sum \frac{x^{3}...

Đã gửi bởi iloveyouproht on 29-07-2016 - 16:55 trong Bất đẳng thức và cực trị

Cho x, y, z>0 :

CMR: $\frac{x^{3}}{(x+y)^{3}}+\frac{y^{3}}{(y+z)^{3}}+\frac{z^{3}}{(z+x)^{3}}\geq \frac{3}{8}$

 

Áp dụng bđt holder ta có : $(\sum \frac{x^{3}}{(x+y)^{3}} )9\geq (\sum \frac{x}{x+y})^{3} => VT\geq \frac{1}{9}(\sum \frac{x}{x+y})^{3} Ta cần cm : \frac{1}{9}(\sum \frac{x}{x+y})^{3} \geq \frac{3}{8} <=> \sum \frac{x}{x+y} \geq \frac{3}{2} Đến đây b tự làm nha$




#648984 Tìm GTNN của $P=\frac{b+2c}{1+a}+\frac...

Đã gửi bởi iloveyouproht on 11-08-2016 - 09:22 trong Bất đẳng thức và cực trị

Ta có đánh giá sau : $9\leq 3c(a+b+c)\leq (4c+a+b)^{2}/4\Rightarrow 2\geq 4c+b+a-4$
Lại có : $\frac {b+2c}{a+1}+\frac {a+2c}{b+1}=(a+b+2c)(\frac {1}{a+1}+\frac{1}{b+1})-2\geq\frac{4(a+b+2c)}{a+b+2}-2 \geq \frac{4(a+b+2c)}{2(a+b+2c)-4}-2$
Đến đây xét hàm f(t) với t=a+b+2c thuộc (0;4] chắc là ổn  :D

Hình Như chỗ này ngược dấu phải k ạ :D




#647046 Tìm GTNN của P=$\frac{1}{2a^{2}+1}+...

Đã gửi bởi iloveyouproht on 29-07-2016 - 15:41 trong Bất đẳng thức và cực trị

B nên xem lại lí thuyết vì dấu "=" $\iff \dfrac{a_1}{b_1}=...\dfrac{a_n}{b_n} \iff 4a^2+2=4b^2+2=6ab \iff a=b=1$

Tks . N nham ty ;)



#660617 cm: $\frac{a^2}{a+b^2} +\frac{b^2}{b+c^2}+\frac{c^2}{c+a^...

Đã gửi bởi iloveyouproht on 04-11-2016 - 21:10 trong Bất đẳng thức và cực trị

Cho các số thưc dương a,b,c thoa man $ a+b+c \geq ab+bc+ac$

cm:

$\frac{a^2}{a+b^2} +\frac{b^2}{b+c^2}+\frac{c^2}{c+a^2} \geq \frac{a+b+c}{2}$

                                           

cho các số thưc $a,b,c abc<0$  va $a+b+c=0$ tìm gtnn  $P =(\frac{1}{a}+\frac{1}{b}+\frac{1}{c})(1-ab-bc-ac) +\frac{12abc-8}{ab+bc+ac}$

Chém tạm bài 1 ạ : 

$\sum \frac{a^{2}}{a+b^{2}}=\sum a-\sum \frac{ab^{2}}{a+b^{2}} \geq \sum a- \sum \frac{ab^{2}}{2b\sqrt{a}} = \sum a - \frac{\sqrt{a}b}{2} \geq \sum a - \frac{\sqrt{(\sum a)(\sum ab)}}{2} \geq \sum a - \frac{\sum a}{2}$ ( Do $ a+b+c \geq ab+bc+ac$ ) => ĐPCM 




#643510 Tìm min $P=\sum\frac{1}{a}+48\sum ab$

Đã gửi bởi iloveyouproht on 03-07-2016 - 21:47 trong Bất đẳng thức và cực trị

PedK72m.png




#647041 Tìm GTNN của P=$\frac{1}{2a^{2}+1}+...

Đã gửi bởi iloveyouproht on 29-07-2016 - 15:30 trong Bất đẳng thức và cực trị

[quote name="leminhnghiatt" post="647035" timestamp="1469780007"]


Ý bạn là đề thế này
Cũng tương tự thôi b



$P=\dfrac{1}{4a^2+2}+\dfrac{1}{4b^2+2}+\dfrac{1}{6ab}+\dfrac{1}{6ab}+\dfrac{6044}{3ab}$

$\geq \dfrac{16}{4a^2+12ab+4b^2+4}+\dfrac{6044}{3ab}$

$=\dfrac{4}{(a+b)^2+ab+1}+\dfrac{6044}{3ab}$

$\geq \dfrac{4}{\dfrac{5}{4}(a+b)^2+1}+\dfrac{6044}{3.\dfrac{(a+b)^2}{4}}=\dfrac{6046}{3}$

Vậy $Min_P=\dfrac{6046}{3} \iff a=b=1
[/quote
ưn



#668887 Tìm min $\sum \frac{x^4}{(x^2+y^2)(x+y)}$

Đã gửi bởi iloveyouproht on 19-01-2017 - 20:51 trong Bất đẳng thức và cực trị

chung minh voi x,y,z la cac so thuc duong sao cho x+y+z=1 tim gia tri nho nhat cua

$\frac{x^4}{(x^2+y^2)(x+y)}+\frac{y^4}{(y^2+z^2)(y+z)}+\frac{z^4}{(z^2+x^2)(z+x)}$ :wacko: :wub: :luoi: :icon6: :icon10:  :icon10:  :icon10:  :wub:  :wub:  :wub:

:$\sum \frac{x^4}{(x+y)(x^2+y^2)}-\sum \frac{y^4}{(x+y)(x^2+y^2)}=\sum x-\sum y=0$

=>$\sum \frac{x^4}{(x+y)(x^2+y^2)}=\frac{1}{2}(\sum \frac{x^4}{(x+y)(x^2+y^2)}+\sum \frac{y^4}{(x+y)(x^2+y^2)})=\frac{1}{2} \sum \frac{x^4+y^4}{(x+y)(x^2+y^2)}\geq \frac       {1}{4}\sum \frac{(x^2+y^2)^2}{(x+y)(x^2+y^2)}\geq \frac{1}{8}\sum \frac{ (x+y)^2}{x+y}=\frac{1}{8}\sum (x+y)=\frac{1}{4}$

=>đpcm




#647054 Tìm min $a^2(a+1) +b^2(b+1)$

Đã gửi bởi iloveyouproht on 29-07-2016 - 17:04 trong Bất đẳng thức và cực trị

Nếu mà tìm max thì dồn biến về b là xong :D




#636090 Chứng minh : $\sqrt{ab} + \sqrt{bc} + \sqrt{ac} \leq...

Đã gửi bởi iloveyouproht on 27-05-2016 - 22:11 trong Bất đẳng thức và cực trị

Cho a,c,b là 3 số thực dương thỏa mãn ab + bc +ac +abc = 4 . Chứng minh :

$\sqrt{ab} + \sqrt{bc} + \sqrt{ac} \leq 3$




#648679 Chứng minh $\LARGE \frac{1}{1+a^3} +...

Đã gửi bởi iloveyouproht on 09-08-2016 - 01:27 trong Bất đẳng thức và cực trị

Cho a,b,c là các số thực dương thỏa mãn abc$\LARGE \geq 1$.CMR

$\LARGE \frac{1}{1+a^3} + \frac{1}{1+b^3} + \frac{1}{1+c^3} \geq \frac{3}{1+abc}$

Trước tiên ta có bđt phụ : $\frac{1}{1+a^{2}} +\frac{1}{1+b^{2}}\geq \frac{2}{1+ab}$ ( Cái này b tự cm , nếu k cm được gì ib mình )

Ta có : $\frac{1}{1+a^{3}} + \frac{1}{1+b^{3}}\geq \frac{2}{1+\sqrt[3]{a^{2}b^{2}}}$

            $\frac{1}{1+c^{3}}+\frac{1}{1+abc}\geq \frac{2}{1+\sqrt[3]{a^{2}b^{2}c^{4}}}$

        Cộng vế theo vế ta được :

 $\sum \frac{1}{1+a^{3}} +\frac{1}{1+abc}\geq 2(\frac{1}{1+\sqrt[3]{a^{2}b^{2}}}+\frac{1}{1+\sqrt[3]{a^{2}b^{2}c^{4}}})\geq 2\frac{2}{1+abc}=\frac{4}{1+abc}$

<=>$\sum \frac{1}{1+a^{3}}\geq \frac{3}{1+abc} ( Q.E.D)$




#660695 $\sum \frac{a}{bc+1}\geq 1$

Đã gửi bởi iloveyouproht on 05-11-2016 - 18:59 trong Bất đẳng thức và cực trị

Cho a,b,c $ \geq 0$ ;a+b+c=1.CMR

$\frac{a}{bc+1}+ \frac{b}{ac+1}+ \frac{c}{ab+1} \geq 1$

Ta có ; $\sum \frac{a}{bc+1} = \sum \frac{a^{2}}{abc+a} \geq \frac{(a+b+c)^{2}}{3abc+\sum a}\geq \frac{(a+b+c)^{2}}{\frac{(a+b+c)^{3}}{9}+\sum a}=\frac{9}{10}$

Uả ủa ủa ????  

 

Sai r T.T




#647447 $(a^{2}+b^{2}+c^{2})^{3} \g...

Đã gửi bởi iloveyouproht on 01-08-2016 - 02:11 trong Bất đẳng thức và cực trị

Ta thuần nhất bất đẳng thức lại dưới dạng

\[(a^{2}+b^{2}+c^{2})^{3} \geqslant 9abc(a^{3}+b^{3}+c^{3}),\]

hay là

\[\sum \left [a^4+11c^4+6b^3c+2ab^3+4ca^3+3c^2a^2+2(a^2-bc)^2+3(b^2-ca)^2+(c^2-ab)^2  \right ](a-b)^2 \geqslant 0.\]

Hiển nhiên đúng.

Ah có thủ thuật gì khi phân tích được ra như thế không ạ




#647093 Tìm min $a^2(a+1) +b^2(b+1)$

Đã gửi bởi iloveyouproht on 29-07-2016 - 20:52 trong Bất đẳng thức và cực trị

Bạn dùng khai triển Abel để phân tích rồi sử dụng giả thiết, ta tìm đuợc Min của biểu thức bằng 48.

max = 48 nha :D

e4KT7Xp.png